LSAT and Law School Admissions Forum

Get expert LSAT preparation and law school admissions advice from PowerScore Test Preparation.

 Administrator
PowerScore Staff
  • PowerScore Staff
  • Posts: 8917
  • Joined: Feb 02, 2011
|
#22908
Complete Question Explanation

Justify the Conclusion-SN. The correct answer choice is (C)

In this stimulus, the author tells us that Upland University computer center has had its board of trustees budget reduced to $1.5 million, although it cannot operate on less than $2.5 million. The author concludes that there is no way that the computer center will remain in operation during the coming year.
  • Premise: The board of trustees only budgeted $1.5 million for the center.

    Premise: The center cannot operate on less than $2.5 million.

    Conclusion: The center cannot stay in operation this coming year.
The conclusion is based on the notion that if the center can't get what it needs from the trustees, it can't stay operational. In order to justify the conclusion in the stimulus, the correct answer choice will somehow reflect that the center has no way to make up this shortfall.

Answer choice (A): This answer choice would not justify the author's conclusion; in fact it would weaken it, providing a possible alternative source of funds for the center.

Answer choice (B): Other programs have nothing to do with the author's conclusion, so this answer choice is incorrect.

Answer choice (C): This is the correct answer choice. If the center has no other source of funds, then it is valid to conclude, based on the budget reduction, that there is no way the center can stay open during the coming year.

Answer choice (D): The author has already established that funds cannot be diverted to the computer center, so the question of whether they can be diverted to other programs is completely irrelevant, providing no support whatsoever for the author's conclusion.

Answer choice (E): The perspectives among the Board, or their motives for reducing the computer center budget, play no role in the argument. The trustees have already set their budget for next year, so the question that remains is whether the center must close as a result of the new reduced budget.
 psik
  • Posts: 6
  • Joined: Jul 13, 2015
|
#19490
I can narrow the answer to choices C) and D), however, at first I thought that D should be right since the conclusion says: "Since the board cannot divert funds from other programs to the computer center, there is no way that the center can be kept operating for the coming year." --> I thought that this new piece of information in the conclusion should be in the answer..

I can only think C) is correct because it guarantees that they won't have anything to go on to operate (but this doesn't lead to the conclusion...).

Thank you
 David Boyle
PowerScore Staff
  • PowerScore Staff
  • Posts: 836
  • Joined: Jun 07, 2013
|
#19501
psik wrote:I can narrow the answer to choices C) and D), however, at first I thought that D should be right since the conclusion says: "Since the board cannot divert funds from other programs to the computer center, there is no way that the center can be kept operating for the coming year." --> I thought that this new piece of information in the conclusion should be in the answer..

I can only think C) is correct because it guarantees that they won't have anything to go on to operate (but this doesn't lead to the conclusion...).

Thank you
Hello psik,

Answer D is not helpful since there may be other sources of funding besides programs at the university (and since you already know that no programs can divert money to the computer center), and the trustees' budget. For example, let's say an imaginary billionaire, Gill Bates, has a habit of donating money to the university every year after the trustees' budget has already been spent.

And that's what answer C takes care of: it ensures that there is no other source of funds available, whether from Gill Bates or anywhere else. (Note that answer C does not mention the "new info in the conclusion" you mention, about diverting money from other programs; but a correct answer does not always need to mention new information, though it must always take that information into account, or at least not contradict it.) And thus does answer C lead successfully to the conclusion, that the center can't operate in the coming year.

Hope this helps,
David
 psik
  • Posts: 6
  • Joined: Jul 13, 2015
|
#19508
Thank you it helps tons.

In retrospect I also see what I did wrong: Answer D is just a simple repetition of the premise "Since the board cannot divert funds from other programs to the computer center"

What I need is A (SInce the board cannot divert funds from other programs to the computer center) + B = The center cannot be kept operating for the coming year

In this case B is answer choices C

If I got this right
 esther913
  • Posts: 23
  • Joined: Apr 13, 2019
|
#64247
Hi,
I am having trouble with (C) and (D).

I thought the conclusion (“...there is no way that the center can be kept operating for the coming year”) was based on the premise that "Since the board cannot divert funds from other programs to the computer center…”
Therefore I thought (D), which states that “No funds from any program at the university can be diverted to other programs”, provides additional information that strengthens the premise. Is (D) wrong because it strengthens the premise instead of the conclusion?

(C) does seem more compelling as it eliminates alternative sources of funding and thus solidifies the conclusion that the center will be unable to operate for the coming year. My concern with (C) is that it seems to work like a defender assumption. I thought defender assumptions were for Assumption type questions only, but maybe that is not the case...

Help would be greatly appreciated!
Thank you.
User avatar
 Dave Killoran
PowerScore Staff
  • PowerScore Staff
  • Posts: 5852
  • Joined: Mar 25, 2011
|
#64319
Hi Esther,

Thanks for the question! It may be that the specific nature of the question caused you some difficulty here. When you talked about (D), you mentioned that you felt that it "provides additional information that strengthens the premise." However, our job here isn't just to strengthen a part of the argument, but rather to fully prove that the conclusion follows from the information we have. Answer choice (D) doesn't prove that conclusion, so it fails there. It's also limited in whether it supports the argument at all since it really is a close restatement of what was said in the stimulus. Repeating a point we already know does not qualify as strengthening on the LSAT! Nor does it in life by the way: have you ever argued with someone who keeps saying the same thing? Their point doesn't get stronger just because they repeat it :-D

With (C), let's analogize the argument to perhaps make it clearer. Imagine for a moment that this argument was about parents giving their teenager money:

  • Teen: Last month my parents gave me $40 for an allowance, but this month they are only giving me $15. That won't work—I need at least $25 a month to get by!

    Parents: Well, we can't get the money from another part of our household budget, so you'll just have to go without this month. Sorry!


    Answer choice (C): The teen has no other source of funds—such as a job or generous friends—that will cover the shortfall this month.
In the above example, you can see how the stimulus isolates the funding to monies provided by the university (or, in my example, the parents). But, if the computer center (or the teen) can get money from another source besides the university (parents), then the argument would fall apart. Answer choice (C) closes that gap and forces the conclusion to be true. Does it work like a Defender? Yes! But it works as an all-powerful Defender ("no source of funds other than those...") eliminating every other source of possible funding, which is why it works here. If it had limited the defense to a single source, such as "The computer center has no wealthy donor who could contribute the funds..." it would then be incorrect (although correct in an Assumption question, which shows the difference).

Please let me know if that helps. Thanks!
 esther913
  • Posts: 23
  • Joined: Apr 13, 2019
|
#64356
That was an awesome explanation! The real life examples were brilliant too :-D
I struggled with this question for a long time, but now I finally understand why (D) is wrong and (C) is correct.
Thank you so much for helping me out. :)

Get the most out of your LSAT Prep Plus subscription.

Analyze and track your performance with our Testing and Analytics Package.